Proving |sinx| =< |x| for all real x.












0












$begingroup$


Everything I did to solve this problem matches the book, except for the last bit.



Use the following to verify the statement $|sin x| leq |x|$.



a) Show that for all $x geq 0$, $f(x)=x-sin x$ is non-decreasing.



$f'(x) = 1-cos x$. Since $-1 leq cos x leq 1$, we have $f'(x) geq 0$ so that $f(x)$ is non-decreasing. Since $f(x)=0$ when $x=0$, we have $f(x)>0$ whenever $x>0$, and hence $sin x leq x$ when $xgeq 0$.



The book then says: And $|sin x|=sin xleq x=|x|$. How can we say that $|sin x|=sin x$?










share|cite|improve this question











$endgroup$












  • $begingroup$
    You can’t because it isn’t true.
    $endgroup$
    – David G. Stork
    Jan 13 at 20:55










  • $begingroup$
    Do you have any idea why the book might say that in this context?
    $endgroup$
    – mXdX
    Jan 13 at 20:55






  • 2




    $begingroup$
    I can only imagine that the author is considering only $x$ between $0$ and $1$, since it is otherwise trivial. It should be explicitly stated though.
    $endgroup$
    – Git Gud
    Jan 13 at 20:56












  • $begingroup$
    The problem says for all x greater than or equal to zero, so I'm not really sure.
    $endgroup$
    – mXdX
    Jan 13 at 20:58






  • 1




    $begingroup$
    Observe it is enough to prove it for $0le xle 1$.
    $endgroup$
    – Bernard
    Jan 13 at 21:14
















0












$begingroup$


Everything I did to solve this problem matches the book, except for the last bit.



Use the following to verify the statement $|sin x| leq |x|$.



a) Show that for all $x geq 0$, $f(x)=x-sin x$ is non-decreasing.



$f'(x) = 1-cos x$. Since $-1 leq cos x leq 1$, we have $f'(x) geq 0$ so that $f(x)$ is non-decreasing. Since $f(x)=0$ when $x=0$, we have $f(x)>0$ whenever $x>0$, and hence $sin x leq x$ when $xgeq 0$.



The book then says: And $|sin x|=sin xleq x=|x|$. How can we say that $|sin x|=sin x$?










share|cite|improve this question











$endgroup$












  • $begingroup$
    You can’t because it isn’t true.
    $endgroup$
    – David G. Stork
    Jan 13 at 20:55










  • $begingroup$
    Do you have any idea why the book might say that in this context?
    $endgroup$
    – mXdX
    Jan 13 at 20:55






  • 2




    $begingroup$
    I can only imagine that the author is considering only $x$ between $0$ and $1$, since it is otherwise trivial. It should be explicitly stated though.
    $endgroup$
    – Git Gud
    Jan 13 at 20:56












  • $begingroup$
    The problem says for all x greater than or equal to zero, so I'm not really sure.
    $endgroup$
    – mXdX
    Jan 13 at 20:58






  • 1




    $begingroup$
    Observe it is enough to prove it for $0le xle 1$.
    $endgroup$
    – Bernard
    Jan 13 at 21:14














0












0








0





$begingroup$


Everything I did to solve this problem matches the book, except for the last bit.



Use the following to verify the statement $|sin x| leq |x|$.



a) Show that for all $x geq 0$, $f(x)=x-sin x$ is non-decreasing.



$f'(x) = 1-cos x$. Since $-1 leq cos x leq 1$, we have $f'(x) geq 0$ so that $f(x)$ is non-decreasing. Since $f(x)=0$ when $x=0$, we have $f(x)>0$ whenever $x>0$, and hence $sin x leq x$ when $xgeq 0$.



The book then says: And $|sin x|=sin xleq x=|x|$. How can we say that $|sin x|=sin x$?










share|cite|improve this question











$endgroup$




Everything I did to solve this problem matches the book, except for the last bit.



Use the following to verify the statement $|sin x| leq |x|$.



a) Show that for all $x geq 0$, $f(x)=x-sin x$ is non-decreasing.



$f'(x) = 1-cos x$. Since $-1 leq cos x leq 1$, we have $f'(x) geq 0$ so that $f(x)$ is non-decreasing. Since $f(x)=0$ when $x=0$, we have $f(x)>0$ whenever $x>0$, and hence $sin x leq x$ when $xgeq 0$.



The book then says: And $|sin x|=sin xleq x=|x|$. How can we say that $|sin x|=sin x$?







proof-explanation






share|cite|improve this question















share|cite|improve this question













share|cite|improve this question




share|cite|improve this question








edited Jan 13 at 21:04









Namaste

1




1










asked Jan 13 at 20:52









mXdXmXdX

898




898












  • $begingroup$
    You can’t because it isn’t true.
    $endgroup$
    – David G. Stork
    Jan 13 at 20:55










  • $begingroup$
    Do you have any idea why the book might say that in this context?
    $endgroup$
    – mXdX
    Jan 13 at 20:55






  • 2




    $begingroup$
    I can only imagine that the author is considering only $x$ between $0$ and $1$, since it is otherwise trivial. It should be explicitly stated though.
    $endgroup$
    – Git Gud
    Jan 13 at 20:56












  • $begingroup$
    The problem says for all x greater than or equal to zero, so I'm not really sure.
    $endgroup$
    – mXdX
    Jan 13 at 20:58






  • 1




    $begingroup$
    Observe it is enough to prove it for $0le xle 1$.
    $endgroup$
    – Bernard
    Jan 13 at 21:14


















  • $begingroup$
    You can’t because it isn’t true.
    $endgroup$
    – David G. Stork
    Jan 13 at 20:55










  • $begingroup$
    Do you have any idea why the book might say that in this context?
    $endgroup$
    – mXdX
    Jan 13 at 20:55






  • 2




    $begingroup$
    I can only imagine that the author is considering only $x$ between $0$ and $1$, since it is otherwise trivial. It should be explicitly stated though.
    $endgroup$
    – Git Gud
    Jan 13 at 20:56












  • $begingroup$
    The problem says for all x greater than or equal to zero, so I'm not really sure.
    $endgroup$
    – mXdX
    Jan 13 at 20:58






  • 1




    $begingroup$
    Observe it is enough to prove it for $0le xle 1$.
    $endgroup$
    – Bernard
    Jan 13 at 21:14
















$begingroup$
You can’t because it isn’t true.
$endgroup$
– David G. Stork
Jan 13 at 20:55




$begingroup$
You can’t because it isn’t true.
$endgroup$
– David G. Stork
Jan 13 at 20:55












$begingroup$
Do you have any idea why the book might say that in this context?
$endgroup$
– mXdX
Jan 13 at 20:55




$begingroup$
Do you have any idea why the book might say that in this context?
$endgroup$
– mXdX
Jan 13 at 20:55




2




2




$begingroup$
I can only imagine that the author is considering only $x$ between $0$ and $1$, since it is otherwise trivial. It should be explicitly stated though.
$endgroup$
– Git Gud
Jan 13 at 20:56






$begingroup$
I can only imagine that the author is considering only $x$ between $0$ and $1$, since it is otherwise trivial. It should be explicitly stated though.
$endgroup$
– Git Gud
Jan 13 at 20:56














$begingroup$
The problem says for all x greater than or equal to zero, so I'm not really sure.
$endgroup$
– mXdX
Jan 13 at 20:58




$begingroup$
The problem says for all x greater than or equal to zero, so I'm not really sure.
$endgroup$
– mXdX
Jan 13 at 20:58




1




1




$begingroup$
Observe it is enough to prove it for $0le xle 1$.
$endgroup$
– Bernard
Jan 13 at 21:14




$begingroup$
Observe it is enough to prove it for $0le xle 1$.
$endgroup$
– Bernard
Jan 13 at 21:14










2 Answers
2






active

oldest

votes


















2












$begingroup$

$|sin x|=sin xleq x=|x|$ is of course wrong. So what is the correct argument?



For $lvert x rvert ge 1$ we trivially have $|sin x| le|x|$.



Moreover we know that $sin xleq x = lvert x rvert$ for $x ge 0$. It remains to consider $-1 le x le 0$. In this range
$$lvert sin x rvert = -sin(x) =sin(-x) le -x = lvert x rvert.$$






share|cite|improve this answer









$endgroup$





















    1












    $begingroup$

    Since both $|sin x|$ and $|x|$ are even functions, you only need to show the statement when $xge0$. Also for $x>1$ the inequality holds true for obvious reasons and in the interval $[0,1]$ we have $$|sin x|=sin x\|x|=x$$






    share|cite|improve this answer









    $endgroup$













      Your Answer





      StackExchange.ifUsing("editor", function () {
      return StackExchange.using("mathjaxEditing", function () {
      StackExchange.MarkdownEditor.creationCallbacks.add(function (editor, postfix) {
      StackExchange.mathjaxEditing.prepareWmdForMathJax(editor, postfix, [["$", "$"], ["\\(","\\)"]]);
      });
      });
      }, "mathjax-editing");

      StackExchange.ready(function() {
      var channelOptions = {
      tags: "".split(" "),
      id: "69"
      };
      initTagRenderer("".split(" "), "".split(" "), channelOptions);

      StackExchange.using("externalEditor", function() {
      // Have to fire editor after snippets, if snippets enabled
      if (StackExchange.settings.snippets.snippetsEnabled) {
      StackExchange.using("snippets", function() {
      createEditor();
      });
      }
      else {
      createEditor();
      }
      });

      function createEditor() {
      StackExchange.prepareEditor({
      heartbeatType: 'answer',
      autoActivateHeartbeat: false,
      convertImagesToLinks: true,
      noModals: true,
      showLowRepImageUploadWarning: true,
      reputationToPostImages: 10,
      bindNavPrevention: true,
      postfix: "",
      imageUploader: {
      brandingHtml: "Powered by u003ca class="icon-imgur-white" href="https://imgur.com/"u003eu003c/au003e",
      contentPolicyHtml: "User contributions licensed under u003ca href="https://creativecommons.org/licenses/by-sa/3.0/"u003ecc by-sa 3.0 with attribution requiredu003c/au003e u003ca href="https://stackoverflow.com/legal/content-policy"u003e(content policy)u003c/au003e",
      allowUrls: true
      },
      noCode: true, onDemand: true,
      discardSelector: ".discard-answer"
      ,immediatelyShowMarkdownHelp:true
      });


      }
      });














      draft saved

      draft discarded


















      StackExchange.ready(
      function () {
      StackExchange.openid.initPostLogin('.new-post-login', 'https%3a%2f%2fmath.stackexchange.com%2fquestions%2f3072502%2fproving-sinx-x-for-all-real-x%23new-answer', 'question_page');
      }
      );

      Post as a guest















      Required, but never shown

























      2 Answers
      2






      active

      oldest

      votes








      2 Answers
      2






      active

      oldest

      votes









      active

      oldest

      votes






      active

      oldest

      votes









      2












      $begingroup$

      $|sin x|=sin xleq x=|x|$ is of course wrong. So what is the correct argument?



      For $lvert x rvert ge 1$ we trivially have $|sin x| le|x|$.



      Moreover we know that $sin xleq x = lvert x rvert$ for $x ge 0$. It remains to consider $-1 le x le 0$. In this range
      $$lvert sin x rvert = -sin(x) =sin(-x) le -x = lvert x rvert.$$






      share|cite|improve this answer









      $endgroup$


















        2












        $begingroup$

        $|sin x|=sin xleq x=|x|$ is of course wrong. So what is the correct argument?



        For $lvert x rvert ge 1$ we trivially have $|sin x| le|x|$.



        Moreover we know that $sin xleq x = lvert x rvert$ for $x ge 0$. It remains to consider $-1 le x le 0$. In this range
        $$lvert sin x rvert = -sin(x) =sin(-x) le -x = lvert x rvert.$$






        share|cite|improve this answer









        $endgroup$
















          2












          2








          2





          $begingroup$

          $|sin x|=sin xleq x=|x|$ is of course wrong. So what is the correct argument?



          For $lvert x rvert ge 1$ we trivially have $|sin x| le|x|$.



          Moreover we know that $sin xleq x = lvert x rvert$ for $x ge 0$. It remains to consider $-1 le x le 0$. In this range
          $$lvert sin x rvert = -sin(x) =sin(-x) le -x = lvert x rvert.$$






          share|cite|improve this answer









          $endgroup$



          $|sin x|=sin xleq x=|x|$ is of course wrong. So what is the correct argument?



          For $lvert x rvert ge 1$ we trivially have $|sin x| le|x|$.



          Moreover we know that $sin xleq x = lvert x rvert$ for $x ge 0$. It remains to consider $-1 le x le 0$. In this range
          $$lvert sin x rvert = -sin(x) =sin(-x) le -x = lvert x rvert.$$







          share|cite|improve this answer












          share|cite|improve this answer



          share|cite|improve this answer










          answered Jan 13 at 22:09









          Paul FrostPaul Frost

          11.6k3935




          11.6k3935























              1












              $begingroup$

              Since both $|sin x|$ and $|x|$ are even functions, you only need to show the statement when $xge0$. Also for $x>1$ the inequality holds true for obvious reasons and in the interval $[0,1]$ we have $$|sin x|=sin x\|x|=x$$






              share|cite|improve this answer









              $endgroup$


















                1












                $begingroup$

                Since both $|sin x|$ and $|x|$ are even functions, you only need to show the statement when $xge0$. Also for $x>1$ the inequality holds true for obvious reasons and in the interval $[0,1]$ we have $$|sin x|=sin x\|x|=x$$






                share|cite|improve this answer









                $endgroup$
















                  1












                  1








                  1





                  $begingroup$

                  Since both $|sin x|$ and $|x|$ are even functions, you only need to show the statement when $xge0$. Also for $x>1$ the inequality holds true for obvious reasons and in the interval $[0,1]$ we have $$|sin x|=sin x\|x|=x$$






                  share|cite|improve this answer









                  $endgroup$



                  Since both $|sin x|$ and $|x|$ are even functions, you only need to show the statement when $xge0$. Also for $x>1$ the inequality holds true for obvious reasons and in the interval $[0,1]$ we have $$|sin x|=sin x\|x|=x$$







                  share|cite|improve this answer












                  share|cite|improve this answer



                  share|cite|improve this answer










                  answered Jan 13 at 22:31









                  Mostafa AyazMostafa Ayaz

                  17k31039




                  17k31039






























                      draft saved

                      draft discarded




















































                      Thanks for contributing an answer to Mathematics Stack Exchange!


                      • Please be sure to answer the question. Provide details and share your research!

                      But avoid



                      • Asking for help, clarification, or responding to other answers.

                      • Making statements based on opinion; back them up with references or personal experience.


                      Use MathJax to format equations. MathJax reference.


                      To learn more, see our tips on writing great answers.




                      draft saved


                      draft discarded














                      StackExchange.ready(
                      function () {
                      StackExchange.openid.initPostLogin('.new-post-login', 'https%3a%2f%2fmath.stackexchange.com%2fquestions%2f3072502%2fproving-sinx-x-for-all-real-x%23new-answer', 'question_page');
                      }
                      );

                      Post as a guest















                      Required, but never shown





















































                      Required, but never shown














                      Required, but never shown












                      Required, but never shown







                      Required, but never shown

































                      Required, but never shown














                      Required, but never shown












                      Required, but never shown







                      Required, but never shown







                      Popular posts from this blog

                      Human spaceflight

                      Can not write log (Is /dev/pts mounted?) - openpty in Ubuntu-on-Windows?

                      File:DeusFollowingSea.jpg